Torts Bar Questions

¡Supera tus tareas y exámenes ahora con Quizwiz!

A defendant, an inexperienced driver, borrowed a car from the plaintiff, a casual acquaintance, for the express purpose of driving it several blocks to the local drug store. Instead, the defendant drove the car, which then was worth $12,000, 100 miles to another city. While the defendant was driving in the other city the next day, the car was hit by a negligently driven truck and sustained damage that will cost $3,000 to repair. If repaired, the car will be fully restored to its former condition. If the plaintiff asserts a claim against the defendant based on conversion, the plaintiff should recover a judgment for A: $12,000. B: $3,000. C: $3,000 plus damages for the loss of the use of the car during its repair. D: nothing, because the defendant was not negligent.

A: $12,000. ----- The plaintiff should recover the fair market value of the car, $12,000, because the defendant substantially interfered with the plaintiff's chattel by driving the car 100 miles, keeping it overnight, and incurring $3,000 in damages. This amounts to a conversion claim, which allows for damages for the full value of the car at the time of the conversion

A plaintiff sustained personal injuries in a three-car collision caused by the concurrent negligence of a trucker and a bus driver. In the plaintiff's action for damages against the other two drivers, the jury apportioned the negligence 30% to the plaintiff, 30% to the trucker, and 40% to the bus driver. The plaintiff's total damages were $100,000. A state statute provides for a system of pure comparative negligence, joint and several liability of concurrent tortfeasors, and contribution based upon proportionate fault. If the plaintiff chooses to pursue the claim against the trucker alone, she will be entitled to collect at most A: $70,000 from the trucker, and then the trucker will be entitled to collect $40,000 from the bus driver. B: $30,000 from the trucker, and then the trucker will be entitled to collect $10,000 from the bus driver. C: $30,000 from the trucker, and then the trucker will be entitled to collect nothing from the bus driver. D: nothing from the trucker, because the percentage of fault for the trucker is not greater than that of the plaintiff.

A: $70,000 from the trucker, and then the trucker will be entitled to collect $40,000 from the bus driver.

A school bus driver reported to a middle school principal that a student had harassed other children on the bus. The principal informed the student's parents of the bus driver's report and told them that, because of the student's behavior, the student could not ride the bus for the next week and would have to be driven to school by a parent. The following Monday morning, after the bus driver had let the children off the bus in front of the school, but before she could close the door and drive away, the student's father pulled his car directly in front of her bus, blocking the driver's path. Because there was another bus right behind hers, the driver was unable to move her bus. The father got out of his car and strode toward the open door of the bus, screaming at the driver: "You messed with the wrong family! I am going to get you!" Feeling threatened, the bus driver quickly closed the door. The father pounded on the door with enough force to dent it, screaming obscenities at the driver, until a school security guard intervened. If the driver were to sue the father, which cause of action would give her the best chance of recovery? A: Assault. B: Battery. C: Intentional infliction of emotional distress. D: Trespass to chattels.

A: Assault.

A patient received anesthesia while giving birth. Upon awakening from the anesthesia, she discovered a severe burn on the inner portion of her right knee. The patient has brought a medical malpractice action in which she has joined all of the physicians and nurses who exercised control over her person, the delivery room, the medical procedures, and the equipment used during the period in which she was unconscious. The defendants have jointly moved for summary judgment. The patient has produced affidavits that establish that the applicable professional standard of care was violated. What would be the patient's best argument against the motion? A: At least one of the defendants had control over whatever agency or instrumentality caused the patient's injury. B: The defendants were acting in concert. C: The patient has produced affidavits that establish that the applicable professional standard of care was violated. D: The patient was in no way responsible for her injury.

A: At least one of the defendants had control over whatever agency or instrumentality caused the patient's injury. --- Showing that a defendant had control over the instrumentality that caused her injury would be a basis for the application of res ipsa loquitur in some jurisdictions

A construction company was digging a trench for a new sewer line in a street in a high-crime neighborhood. During the course of the construction, there had been many thefts of tools and equipment from the construction area. One night, the construction company's employees neglected to place warning lights around the trench. A delivery truck drove into the trench and broke an axle. While the truck driver was looking for a telephone to call a tow truck, thieves broke into the truck and stole $350,000 worth of goods. The delivery company sued the construction company to recover for the $350,000 loss and for the damage to its truck. The construction company has stipulated that it was negligent in failing to place warning lights around the trench and admits liability for damage to the truck, but it denies liability for the loss of the goods. On cross-motions for summary judgment on the claim for the goods, how should the court rule? A: Deny both motions, because there is evidence to support a finding that the construction company should have realized that its negligence could create an opportunity for a third party to commit a crime. B: Grant the construction company's motion, because no one could have foreseen that the failure to place warning lights could result in the loss of a cargo of valuable goods. C: Grant the construction company's motion, because the criminal acts of third persons were a superseding cause of the loss. D: Grant the delivery company's motion, because but for the construction company's actions, the goods would not have been stolen.

A: Deny both motions, because there is evidence to support a finding that the construction company should have realized that its negligence could create an opportunity for a third party to commit a crime. ----- it had knowledge of regular thefts in the area, which means that a jury could reasonably decide that it was foreseeable.

A longshoreman fell to his death through an open hatch on the deck of a ship. The longshoreman was an employee of a company that had contracted with the ship's owner to load and unload the ship. The fall occurred at night, when loading work was over for the day, and there was no reason for the longshoreman to have been near the hatch. A negligence action was filed against the ship's owner for the death of the longshoreman. In that action, the owner has moved for summary judgment and has provided unrebutted evidence that it is customary for the crews of ships to open the hatches for ventilation after the longshoremen have left the ships. How should the court respond to the motion? A: Deny the motion and submit the case to the jury with instructions that the custom is relevant but not conclusive on the issue of negligence. B: Deny the motion and submit the case to the jury with instructions that the ship's owner should win if the longshoreman was improperly near the hatch. C: Deny the motion, because the probability of serious injury caused by falling down an open hatch clearly outweighs the burden of keeping the hatch closed. D: Grant the motion, because the custom should be considered conclusive on the issue of negligence

A: Deny the motion and submit the case to the jury with instructions that the custom is relevant but not conclusive on the issue of negligence. ---- The motion should be denied because, although it is custom for the hatches to be left open for ventilation, it cannot be said to be conclusive on the issue of negligence. There are not enough facts in this question to determine if that alone absolves the ship's owner of liability

An employer retained a doctor to evaluate medical records of prospective employees. The doctor informed the employer that an applicant, a prospective employee, suffered from AIDS. The employer informed the applicant of this and declined to hire her. The applicant was shocked by this news and suffered a heart attack as a result. Subsequent tests revealed that the applicant in fact did not have AIDS. The doctor had negligently confused the applicant's file with that of another prospective employee. If the applicant sued the doctor for damages, on which of the following causes of action would the applicant recover? A: Negligent infliction of emotional distress. B: Invasion of privacy. C: Negligent misrepresentation. D: Both invasion of privacy and negligent misrepresentation

A: Negligent infliction of emotional distress.

A defendant has a small trampoline in his backyard which, as he knows, is commonly used by neighbor children as well as his own. The trampoline is in good condition, is not defective in any way, and normally is surrounded by mats to prevent injury if a user should fall off. Prior to leaving with his family for the day, the defendant leaned the trampoline up against the side of the house and placed the mats in the garage. While the defendant and his family were away, the plaintiff, aged 11, a new boy in the neighborhood, wandered into the defendant's yard and saw the trampoline. The plaintiff had not previously been aware of its presence, but, having frequently used a trampoline before, he decided to set it up, and started to jump. He lost his balance on one jump and took a hard fall on the bare ground, suffering a serious injury that would have been prevented by the mats. An action has been brought against the defendant on the plaintiff's behalf to recover damages for the injuries the plaintiff sustained from his fall. In this jurisdiction, the traditional common-law rules pertaining to contributory negligence have been replaced by a pure comparative negligence rule. In his action against the defendant, will the plaintiff prevail? A: No, because children likely to be attracted by the trampoline would normally realize the risk of using it without mats. B: No, because the plaintiff failed to exercise reasonable care commensurate with his age, intelligence, and experience. C: No, because the plaintiff entered the defendant's yard and used the trampoline without the defendant's permission. D: No, because the plaintiff did not know about the trampoline before entering the defendant's yard and thus was not "lured" onto the premises

A: No, because children likely to be attracted by the trampoline would normally realize the risk of using it without mats.

An 11-year-old boy was driving a full-size motorcycle on a private road, where the boy was a trespasser. The motorcycle hit a tire that had fallen off a truck driven by a delivery company employee who was making a delivery to an address on the private road. The boy was injured when his motorcycle went out of control after striking the tire. In a negligence action brought on behalf of the boy against the delivery company, the company contends that the boy was contributorily negligent and that his damages, if any, should be reduced in conformance with the jurisdiction's comparative negligence statute. The boy argues that his conduct should be judged according to the standard of a reasonable child of like age, intelligence, and experience under the circumstances. Is the boy entitled to be judged according to the standard of care that he has argued for? A: No, because the boy was driving a motorcycle. B: No, because the boy was trespassing on the private road. C: Yes, because comparative negligence applies. D: Yes, because the boy was 11 years old at the time

A: No, because the boy was driving a motorcycle.

An associate professor in the pediatrics department of a local medical school was denied tenure. He asked a national education lobbying organization to represent him in his efforts to have the tenure decision reversed. In response to a letter from the organization on the professor's behalf, the dean of the medical school wrote to the organization explaining truthfully that the professor had been denied tenure because of reports that he had abused two of his former patients. Several months later, after a thorough investigation, the allegations were proven false, and the professor was granted tenure. He had remained working at the medical school at full pay during the tenure decision review process and thus suffered no pecuniary harm. In a suit for libel by the professor against the dean of the medical school, will the professor be likely to prevail? A: No, because the professor invited the libel. B: No, because the professor suffered no pecuniary loss. C: Yes, because the dean had a duty to investigate the rumor before repeating it. D: Yes, because the dean's defamatory statement was in the form of a writing

A: No, because the professor invited the libel. ---- the dean would be able to raise an absolute privilege of consent given that the defamatory statement was in response to the initial letter from the organization acting on the professor's behalf

An eight-year-old child went to the grocery store with her mother. The child pushed the grocery cart while her mother put items into it. The child's mother remained near the child at all times. Another customer in the store noticed the child pushing the cart in a manner that caused the customer no concern. A short time later, the cart the child was pushing struck the customer in the knee, inflicting serious injury. If the customer brings an action, based on negligence, against the child, the child's best argument in defense would be that A: The child exercised care commensurate with her age, intelligence, and experience. B: The child is not subject to tort liability. C: The child was subject to parental supervision. D: The customer assumed the risk that the child might hit the customer with the cart

A: The child exercised care commensurate with her age, intelligence, and experience. ----- The customer's claim for negligence will be allowed, but the child will only be held to the standard of care expected of "a reasonable child" of the same age, training, maturity, experience, and intelligence.

In a civil action, a plaintiff sued a decedent's estate to recover damages for injuries she suffered in a collision between her car and one driven by the decedent. At trial, the plaintiff introduced undisputed evidence that the decedent's car had swerved across the centerline of the highway into oncoming traffic, where it had collided with the plaintiff's car. The decedent's estate introduced undisputed evidence that, before he swerved across the centerline, the decedent had suffered a fatal heart attack, which he had no reason to foresee, and that, just prior to the heart attack, the decedent had been driving at a reasonable speed and in a reasonable manner. A statute makes it a traffic offense to cross the centerline of a highway. In this case, which party is likely to prevail? A: The decedent's estate, because its rebuttal evidence is undisputed. B: The decedent's estate, because the plaintiff has not established a prima facie case of liability. C: The plaintiff, because the accident was of a type that does not ordinarily happen in the absence of negligence on the actor's part. D: The plaintiff, because the decedent crossed the centerline in violation of the statute.

A: The decedent's estate, because its rebuttal evidence is undisputed. --- The plaintiff's evidence that the decedent violated the statute and crossed over the centerline establishes a prima facie case of negligence. However, the prima facie case of negligence may be rebutted by showing that compliance with the statute was beyond the defendant's control.

A man's father died in a hospital. The hospital maintains a morgue with refrigerated drawers a bit larger than the human body. The decedent's body was placed in such a drawer awaiting pickup by a mortician. Before the mortician called for the body, a hospital orderly placed two opaque plastic bags in the drawer with the decedent's body. One bag contained the decedent's personal effects, and the other contained an amputated leg from some other hospital patient. It is stipulated that the hospital was negligent to allow the amputated leg to get into the decedent's drawer. The mortician delivered the two opaque plastic bags to the man, assuming both contained personal effects. The man was shocked when he opened the bag containing the amputated leg. The man sued the hospital to recover for emotional distress. At the trial, the man testified that the experience had been extremely upsetting, that he had had recurring nightmares about it, and that his family and business relationships had been adversely affected for a period of several months. He did not seek medical or psychiatric treatment for his emotional distress. Who should prevail? A: The man, because of the sensitivity people have regarding the care of the bodies of deceased relatives. B: The man, because hospitals are strictly liable for mishandling dead bodies. C: the hospital, because the man did not require medical or psychiatric treatment. D: the hospital, because the man suffered no bodily harm.

A: The man, because of the sensitivity people have regarding the care of the bodies of deceased relatives.

For 10 years, a vacationer and a neighbor have owned summer vacation homes on adjoining lots. A stream flows through both lots. As a result of a childhood swimming accident, the vacationer is afraid of water and has never gone close to the stream. The neighbor built a dam on her property that has completely stopped the flow of the stream to the vacationer's property. The dam unreasonably interferes with the use and enjoyment of the vacationer's property but was built in conformity with all applicable laws. In a suit by the vacationer against the neighbor, will the vacationer prevail? A: Yes, because the damming unreasonably interferes with the use and enjoyment of the vacationer's property. B: Yes, because the neighbor intended to affect the vacationer's property. C: No, because the vacationer made no use of the stream. D: No, because the dam was built in conformity with all applicable laws.

A: Yes, because the damming unreasonably interferes with the use and enjoyment of the vacationer's property. --- nuisance. A landowner who causes a substantial, unreasonable interference with a neighbor's use or enjoyment of his property without a valid defense is liable for private nuisance. This rule also applies to flowing water, so an upstream owner may not stop the flow of water to a downstream property if it would substantially interfere with the use and enjoyment of the downstream property.

A farmer owns a small farm with several head of cattle which are kept in a fenced grazing area. One day the cattle were frightened by a thunderstorm, an occasional occurrence in the area. The cattle broke through the fence, entered onto the neighbor's property, and severely damaged the neighbor's crops. Because the farmer's cattle had panicked during past thunderstorms, the farmer had been diligent in maintaining the fence. Under the law of the state, landowners are not required to erect fences to prevent the intrusion of livestock. If the neighbor sues the farmer to recover for the damage done to his crops, will the neighbor prevail? A: Yes, because the farmer's cattle caused the damage to the neighbor's crops. B: Yes, because the farmer's cattle had panicked during previous thunderstorms. C: No, because the fence was not negligently maintained by the farmer. D: No, because the thunderstorm was a force of nature.

A: Yes, because the farmer's cattle caused the damage to the neighbor's crops. and C: No, because the fence was not negligently maintained by the farmer.

The defendant operates a collection agency. He was trying to collect a valid $400 bill for medical services rendered to the plaintiff by a doctor that was past due. The defendant went to the plaintiff's house and when the plaintiff's mother answered the door, the defendant told her that he was there to collect a bill owed by the plaintiff. The mother told the defendant that because of the plaintiff's illness, the plaintiff had been unemployed for six months, that she was still ill and unable to work, and that she would pay the bill as soon as she could. The defendant, in a loud voice, demanded to see the plaintiff and said that if he did not receive payment immediately, he would file a criminal complaint charging her with fraud. The plaintiff, hearing the conversation, came to the door. The defendant, in a loud voice, repeated his demand for immediate payment and his threat to use criminal process. Assume that the plaintiff did not suffer physical harm as a result of the defendant's conduct, but did suffer severe emotional distress. If the plaintiff asserts a claim against the defendant based on intentional infliction of emotional distress, will the plaintiff prevail? A: Yes, because the plaintiff suffered severe emotional distress as a result of the defendant's conduct. B: No, because the bill for medical services was valid and past due. C: No, because the plaintiff did not suffer physical harm as a result of the defendant's conduct. D: No, because the defendant's conduct created almost no risk of physical harm to the plaintiff

A: Yes, because the plaintiff suffered severe emotional distress as a result of the defendant's conduct. ---- a bill collector, came to the house of the plaintiff, a severely ill person, and loudly threatened her multiple times with criminal fraud charges over the payment of a hospital bill. This would likely constitute extreme and outrageous behavior.

The police in a large city notified local gas station attendants that a woman recently had committed armed robberies at five city gas stations. The police said that the woman was approximately 75 years old, had white hair, and drove a vintage, cream-colored Ford Thunderbird. Attendants were advised to call the police if they saw her, but to not attempt to apprehend her. Armed robbery is a felony under state law. A traveler was passing through the city on a cross-country journey. The traveler was a 75-year-old woman who had white hair and drove a vintage, cream-colored Ford Thunderbird. When the traveler drove into a gas station, the owner of the station thought the traveler must be the robber wanted by the police. After checking the oil at the traveler's request, the owner falsely informed the traveler that she had a broken fan belt, that her car could not be driven without a new belt, that it would take him about an hour to replace it, and that she should stay in his office for consultation about the repair. The traveler was greatly annoyed that her journey was delayed, but she stayed in the owner's office while she waited for her car. The owner telephoned the police and, within the hour, the police came and questioned the traveler. The police immediately determined that the traveler was not the woman, and the traveler resumed her journey without further delay. In the traveler's action for false imprisonment against the owner, the traveler will A: not prevail, because the owner reasonably believed that the traveler was the wanted woman. B: not prevail, because the traveler suffered no physical or mental harm. C: prevail, because the traveler reasonably believed she could not leave the owner's premises. D: prevail, because the owner lied to the traveler about the condition of her car

A: not prevail, because the owner reasonably believed that the traveler was the wanted woman. ---- the owner will be protected by privilege given that he reasonably believed the traveler had committed a felony, which had actually occurred, and the scope of the confinement was not unreasonable

A supermarket is in a section of town where there are sometimes street fights and where pedestrians are occasionally the victims of pickpockets and muggers. In recognition of the unusual number of robberies in the area, the supermarket posted signs in the store that read: "Warning: There are pickpockets and muggers at work in this part of the city. The supermarket is not responsible for the acts of criminals." Other than posting the signs, the supermarket took no other precautions to prevent criminal activity on the premises. One evening, a customer drove to the supermarket to see about a special on turkeys that the supermarket was advertising. She decided that the turkeys were too large and left the store without purchasing anything. In the parking lot, she was attacked by an unknown man who raped her and then ran away. If the customer sues the supermarket, the result should be for the A: plaintiff, because the supermarket failed to take reasonable steps to protect customers against criminal attack in its parking lot. B: plaintiff, because the supermarket is liable for harm to business invitees on its premises. C: defendant, because the warning signs were visible to the customer. D: defendant, because the rapist was the proximate cause of the customer's injuries.

A: plaintiff, because the supermarket failed to take reasonable steps to protect customers against criminal attack in its parking lot.

A college student purchased a large bottle of No-Flake dandruff shampoo, manufactured by a shampoo company. The box containing the bottle stated in part: "CAUTION - Use only one capful at most once a day. Greater use may cause severe damage to the scalp." The college student read the writing on the box, removed the bottle, and threw the box away. The college student's roommate asked to use the No-Flake, and the college student said, "Be careful not to use too much." The roommate thereafter used No-Flake twice a day, applying two or three capfuls each time, notwithstanding the label statement that read: "Use no more than one capful per day. See box instructions." The more he used No-Flake, the more inflamed his scalp became, the more it itched, and the more he used. After three weeks of such use, the roommate finally consulted a doctor who diagnosed his problem as a serious and irreversible case of dermatitis caused by excessive exposure to the active ingredients by No-Flake. These ingredients are uniquely effective at controlling dandruff, but there is no way to remove a remote risk to a small percentage of persons who may contract dermatitis as the result of applying, for prolonged periods of time, amounts of No-Flake substantially in excess of the directions. This jurisdiction adheres to the traditional common-law rules pertaining to contributory negligence and assumption of risk. If the roommate asserts a claim against the college student for his dermatitis injuries, the college student's best defense will be that A: the roommate was contributorily negligent. B: the roommate assumed the risk. C: the college student had no duty toward the roommate, who was a gratuitous donee. D: the college student had no duty toward

A: the roommate was contributorily negligent. --- The roommate was negligent in not heeding the college student's warning to be careful and not use too much of the shampoo, so, under common law contributory negligence, the roommate will not be able to recover anything from the college student

The plaintiff was a passenger in a car that was struck in the rear by a car driven by a student. The collision resulted from the student's negligence in failing to keep a proper lookout. The plaintiff's physician found that the collision had aggravated a mild osteoarthritic condition in her lower back and had brought on similar, but new, symptoms in her neck and upper back. Six months after the first accident, the plaintiff was a passenger in a car that was struck in the rear by a car driven by a doctor. The collision resulted from the doctor's negligence in failing to keep a proper lookout. The plaintiff's physician found that the second collision had caused a general worsening of the plaintiff's condition, marked by a significant restriction of movement and muscle spasms in her back and neck. The physician believes the plaintiff's worsened condition is permanent, and he can find no basis for apportioning responsibility for her present worsened condition between the two automobile collisions. The plaintiff brought an action for damages against the student and the doctor. At the close of the plaintiff's evidence, as outlined above, each of the defendants moved for a directed verdict in his favor on the ground that the plaintiff had failed to produce evidence on which the jury could determine how much damage each defendant had caused. The jurisdiction adheres to the common law rules regarding joint and several liability. The plaintiff's best argument in opposition to the defendants' motions would be that the defendants are jointly and severally liable for the plaintiff's entire harm, because A: the wrongdoers, rather than their victim, should bear the burden of the impossibility of apportionment. B: the defendants breached a common duty that each of them owed to the plaintiff. C: each of the defendants was the proximate cause in fact of all of the plaintiff's damages. D: the defendants are joint tortfeasors who aggravated the plaintiff's preexisting condition.

A: the wrongdoers, rather than their victim, should bear the burden of the impossibility of apportionment. ----- Where the requirement of actual proof under these facts would result in a harsh result on an innocent victim, courts have traditionally held the defendants to be jointly and severally liable for the cause-in-fact, considering the injury to be indivisible as a matter of policy

A 14-year-old girl of low intelligence received her parents' permission to drive their car. She had had very little experience driving a car and did not have a driver's license. Although she did the best she could, she lost control of the car and hit a pedestrian. The pedestrian has brought a negligence action against the girl. Is the pedestrian likely to prevail? A: No, because only the girl's parents are subject to liability. B: No, because the girl was acting reasonably for a 14-year-old of low intelligence and little driving experience. C: Yes, because the girl was engaging in an adult activity. D: Yes, because the girl was not old enough to obtain a driver's license

C: Yes, because the girl was engaging in an adult activity. ----- The girl was engaging in a dangerous activity that is characteristically undertaken by adults, so she will be held to the adult standard of care

A driver negligently ran into a pedestrian who was walking along a road. The pedestrian sustained an injury to his knee, causing it to buckle from time to time. Several months later, the pedestrian sustained an injury to his shoulder when his knee buckled, causing him to fall down a flight of stairs. The pedestrian then brought an action against the driver for the injuries to his knee and shoulder. In his action against the driver, for which of his injuries may the pedestrian recover damages? A: For the injuries to his knee and shoulder, because the driver takes the victim as he finds him. B: For the injuries to his knee and shoulder, if the jury finds that the pedestrian's fall down a flight of stairs was a normal consequence of his original injury. C: For the injury to his knee only, because the injury to the pedestrian's shoulder is separable. D: For the injury to his knee only, if the jury finds that the driver could not have foreseen that his negligent driving would cause the pedestrian to fall down a flight of stairs.

B: For the injuries to his knee and shoulder, if the jury finds that the pedestrian's fall down a flight of stairs was a normal consequence of his original injury. --- The jury's finding that the fall was a normal consequence of the original injury will allow for the analysis to proceed to a determination of the scope of recoverable damages. Negligent defendants are liable for the full extent damages, regardless of foreseeability

A patient in a hospital was placed in a wheelchair with his broken leg extended straight out in front of him. As a nurse employed by the hospital was pushing the wheelchair through a set of automatic doors at a normal pace, the doors closed on the patient's foot, injuring it. The nurse attempted to pull the wheelchair back through the doors. This action caused the doors to close more tightly on the patient's foot, injuring it further. The patient sued the hospital, alleging improper maintenance of the doors. The patient has produced no evidence of specific conduct or neglect on the part of the hospital that would have caused the automatic doors to malfunction. The hospital has moved for summary judgment. Should the court grant the hospital's motion? A: No, because a jury could find that there was a latent defect in the doors. B: No, because a jury could find the hospital liable for negligence based on res ipsa loquitur. C: Yes, because proof of an accident, by itself, does not establish that an injured person was a victim of negligence. D: Yes, because the nurse's action was a superseding cause of the injury.

B: No, because a jury could find the hospital liable for negligence based on res ipsa loquitur. ---- There is sufficient evidence here for a res ipsa loquitur showing, meaning that a jury could infer that the hospital was probably negligent.

A bank vice president took substantial kickbacks to approve certain loans that later proved worthless. Upon learning of the kickbacks, the bank's president fired the vice president, telling him, "If you are not out of this bank in 10 minutes, I will have the guards physically throw you out." The vice president left at once. If the vice president asserts a claim against the president based on assault, will the vice president prevail? A: No, because the guards never touched the vice president. B: No, because the president gave the vice president 10 minutes to leave. C: Yes, because the president intended to cause the vice president severe emotional distress. D: Yes, because the president threatened the vice president with a harmful or offensive bodily contact.

B: No, because the president gave the vice president 10 minutes to leave.

A hiker sustained a head injury when he was struck by a limb that fell from a tree. At the time of his injury, the hiker was walking through a forest on private property without the property owner's knowledge or permission. It was determined that the limb fell because the tree was infested with termites. In an action by the hiker against the property owner to recover for his head injury, will the hiker be likely to prevail? A: No, because the property owner could not foresee that anyone would be injured. B: No, because the property owner breached no duty to the hiker, who was a trespasser. C: Yes, because the property owner had a duty to prevent the trees on his property from becoming dangerous. D: Yes, because the property owner is liable for hidden dangers on his property.

B: No, because the property owner breached no duty to the hiker, who was a trespasser. ---- A possessor of land is not required to exercise reasonable care to make his land safe for trespassers.

During a comprehensive evaluation of an adult patient's psychiatric condition, a psychiatrist failed to diagnose the patient's suicidal state. One day after the misdiagnosis, the patient committed suicide. The patient's father, immediately after having been told of his son's suicide, suffered severe emotional distress, which resulted in a stroke. The patient's father was not present at the patient's appointment with the psychiatrist, nor did he witness the suicide. The father has brought an action against the psychiatrist to recover for his severe emotional distress and the resulting stroke. Is the father likely to prevail? A: No, because the father did not sustain a physical impact. B: No, because the psychiatrist's professional duty did not extend to the harms suffered by the patient's father. C: Yes, because the father was a member of the patient's immediate family. D: Yes, because the psychiatrist reasonably could have foreseen that a misdiagnosis would result in the patient's suicide and the resulting emotional distress of the patient's father.

B: No, because the psychiatrist's professional duty did not extend to the harms suffered by the patient's father. --- Courts do not impose a duty on medical professionals to warn third parties when a patient is a danger to himself and not others and will deny liability to family members who suffered emotionally in the event that the patient commits suicide

A 15-year-old boy was killed during a gang fight. Two days after his funeral, the boy's mother saw a television program about gang violence and was shocked to see video of herself weeping over the boy's body. The video had been shot by the television reporting team while the boy's body was still lying on a public street. The mother suffered severe emotional distress as a result of seeing the video. If the mother sues the television station for invasion of her privacy and that of her son, will the mother be likely to prevail? A: No, because a person has no right to privacy after his or her death. B: No, because the street was open to the public and the subject was newsworthy. C: Yes, because the mother did not give permission to have the video used in the program. D: Yes, because the mother suffered severe emotional distress as a result of viewing the video.

B: No, because the street was open to the public and the subject was newsworthy.

A vintner is the owner of a large vineyard and offers balloon rides to visitors who wish to tour the grounds from the air. During one of the rides, the vintner was forced to make a crash landing on his own property due to high winds. Without the vintner's knowledge or consent, a trespasser had entered the vineyard to camp for a couple of days. The trespasser was injured when he was hit by the basket of the descending balloon. If the trespasser sues the vintner to recover damages for his injuries, will the trespasser prevail? A: No, because there is no evidence that the crash landing was made necessary by the vintner's negligence. B: No, because the vintner was unaware of the trespasser's presence until after the injury had occurred. C: Yes, because even a trespasser may recover for injuries caused by an abnormally dangerous activity. D: Yes, because the accident occurred at a place which the vintner knew was frequented by intruders.

B: No, because the vintner was unaware of the trespasser's presence until after the injury had occurred.

A customer fell and injured himself when he slipped on a banana peel while shopping at a grocery store. The banana peel was fresh and unblemished except for a mark made by the heel of the customer's shoe. In an action brought by the customer against the store, these are the only facts in evidence. Should the trial judge permit the case to go to the jury? A: No, because the customer had an obligation to watch where he stepped. B: No, because there is not a reasonable basis for inferring that the store knew or should have known of the banana peel. C: Yes, because it is more likely than not that the peel came from a banana offered for sale by the grocer. D: Yes, because the store could foresee that a customer might slip on a banana peel.

B: No, because there is not a reasonable basis for inferring that the store knew or should have known of the banana peel. ------ Unlike slip-and-fall cases in which res ipsa loquitur is appropriate, the condition of the banana peel does not indicate that it has been on the ground for any significant period of time. Therefore, there is not enough evidence to support a jury verdict that the store staff was negligent in failing to remove it before the customer's fall.

In a plaintiff's action for battery, the evidence established the following: the plaintiff was bad-tempered and, as the defendant knew, carried a gun and used it often; the plaintiff struck the defendant first; during the altercation, the plaintiff repeatedly tried to get to his gun; and the blows inflicted upon the plaintiff by the defendant resulted in the plaintiff being hospitalized. Which finding of fact would be most likely to result in a verdict for the defendant? A: The defendant used no more force than he actually believed was necessary to protect himself against death or serious bodily harm. B: The defendant used no more force than he reasonably believed was necessary to protect himself against death or serious bodily harm. C: The defendant, in fact, feared death or serious bodily harm. D: The defendant was justified in retaliating against the plaintiff because the plaintiff struck the first blow

B: The defendant used no more force than he reasonably believed was necessary to protect himself against death or serious bodily harm.

The day after a seller completed the sale of his house and moved out, one of the slates flew off the roof during a windstorm. The slate struck a pedestrian who was on the public sidewalk. The pedestrian was seriously injured. The roof is old and has lost several slates in ordinary windstorms on other occasions when the seller was present. The pedestrian was also aware that past windstorms had blown slates off the roof. If the pedestrian sues the seller to recover damages for his injuries, will the pedestrian prevail? A: Yes, because the roof was defective when the seller sold the house. B: Yes, because the seller should have been aware of the condition of the roof and should have realized that it was dangerous to persons outside the premises. C: No, because the seller was neither the owner nor the occupier of the house when the pedestrian was injured. D: No, because the pedestrian knew that in the past slates had blown off the roof during windstorms.

B: Yes, because the seller should have been aware of the condition of the roof and should have realized that it was dangerous to persons outside the premises. --- The seller had a duty of reasonable care for the safety of those outside the land to prevent harm resulting from conditions on the land. The facts indicate that ordinary windstorms had previously caused tile loss. Despite the fact that the seller just sold the home, he was or should have been aware of the dangerous condition of the roof and the foreseeability of harm to persons outside the premises from windswept falling tiles.

A defendant and a group of his friends are fanatical basketball fans who regularly meet at each others' houses to watch basketball games on television. Some of the group are fans of the home team, and others are fans of the rival team. When the group has watched televised games between these two teams, fights sometimes have broken out among the group. Despite this fact, the defendant invited the group to his house to watch a championship game between the home team and the rival team. During the game, the defendant's guests became rowdy and antagonistic. Fearing that they would begin to fight, and that a fight would damage his possessions, the defendant asked his guests to leave. They refused to go and soon began to fight. The defendant called the police, and a police officer was sent to the defendant's home. The officer sustained a broken nose in his efforts to stop the fighting. The officer brought an action against the defendant alleging that the defendant was negligent in inviting the group to his house to watch this championship game. The defendant has moved to dismiss the complaint. The best argument in support of this motion would be that A: a rescuer injured while attempting to avert a danger cannot recover damages from the endangered person. B: a police officer is not entitled to a recovery based upon the negligent conduct that created the need for the officer's professional intervention. C: as a matter of law, the defendant's conduct was not the proximate cause of the officer's injury. D: the defendant did not owe the officer a duty to use reasonable care, because the officer was a mere licensee on the defendant's property.

B: a police officer is not entitled to a recovery based upon the negligent conduct that created the need for the officer's professional intervention.

While a driver was taking a leisurely spring drive, he momentarily took his eyes off the road to look at some colorful trees in bloom. As a result, his car swerved a few feet off the roadway, directly toward a pedestrian, who was standing on the shoulder of the road waiting for a chance to cross. When the pedestrian saw the car bearing down on him, he jumped backwards, fell, and injured his knee. The pedestrian sued the driver for damages, and the driver moved for summary judgment. The foregoing facts are undisputed. The driver's motion should be A: denied, because the record shows that the pedestrian apprehended an imminent, harmful contact, with the driver's car. B: denied, because a jury could find that the driver negligently caused the pedestrian to suffer a legally compensable injury. C: granted, because the proximate cause of the pedestrian's injury was his own voluntary act. D: granted, because it is not unreasonable for a person to be distracted momentarily.

B: denied, because a jury could find that the driver negligently caused the pedestrian to suffer a legally compensable injury.

A defendant negligently caused a fire in his house, and the house burned to the ground. As a result, the sun streamed into the plaintiff's yard next door, which previously had been shaded by the defendant's house. The sunshine destroyed some delicate and valuable trees in the plaintiff's yard that could grow only in the shade. The plaintiff has brought a negligence action against the defendant for the loss of the plaintiff's trees. The defendant has moved to dismiss the complaint. The best argument in support of this motion would be that A: the defendant's negligence was not the active cause of the loss of the plaintiff's trees. B: the defendant's duty to avoid the risks created by a fire did not encompass the risk that sunshine would damage the plaintiff's trees. C: the loss of the trees was not a natural and probable consequence of the defendant's negligence. D: the plaintiff suffered a purely economic loss, which is not compensable in a negligence action

B: the defendant's duty to avoid the risks created by a fire did not encompass the risk that sunshine would damage the plaintiff's trees. --- When the fire was negligently started in his own home, the defendant's duty to avoid foreseeable risks posed by a fire arguably would not have included sun damage to his neighbor's trees.

A driver negligently drove his car into a pedestrian, breaking her leg. The pedestrian's leg was put in a cast, and she used crutches to get around. While shopping at her local supermarket, the pedestrian non-negligently placed one of her crutches on a banana peel that had been negligently left on the floor by the manager of the supermarket's produce department. The pedestrian's crutch slipped on the peel, and she fell to the floor, breaking her arm. Had the pedestrian stepped on the banana peel at a time when she did not have to use crutches, she would have regained her balance. The pedestrian sued the driver and the supermarket for her injuries. The pedestrian will be able to recover from A: the driver, for her broken leg only. B: the driver, for both of her injuries. C: the supermarket, for both of her injuries. D: the driver, for her broken leg only, and the supermarket, for her broken arm only

B: the driver, for both of her injuries. ----- The pedestrian's broken arm is proximately connected to the driver's negligent driving. Courts generally hold that subsequent ordinary negligence actions, which may add to the plaintiff's injuries, are a foreseeable consequence of the original action and will not break the chain of causation.

While an equestrian was riding her horse on what she thought was a public path, the owner of a house next to the path approached her, shaking a stick and shouting, "Get off my property." Unknown to the equestrian, the path on which she was riding crossed the private property of the shouting owner. When the equestrian explained that she thought the path was a public trail, the man cursed her, approached the equestrian's horse, and struck the horse with the stick. As a result of the blow, the horse reared, causing the equestrian to fear that she would fall. However, the equestrian managed to stay on her horse, and then departed. Neither the equestrian nor the horse suffered bodily harm. If the equestrian brings an action for damages against the property owner, the result should be for A: the equestrian, for trespass to her chattel property. B: the equestrian, for battery and assault. C: the defendant, because the equestrian suffered no physical harm. D: the defendant, because he was privileged to exclude trespassers from his property

B: the equestrian, for battery and assault.

A plaintiff sustained personal injuries in a three-vehicle collision caused by the concurrent negligence of all three drivers. In the plaintiff's action for damages against the other two drivers, the jury apportioned the negligence 30% to the plaintiff, 30% to a truck driver, and 40% to a cab driver. The plaintiff's total damages were $100,000. Assume that the state has retained the common-law rule pertaining to contribution and that the state's comparative negligence statute provides for a system of pure comparative negligence but abolishes joint and several liability. If the plaintiff chooses to pursue the claim against the truck driver alone, she will be entitled to collect at most A: $70,000 from the truck driver, and then the truck driver will be entitled to collect $40,000 from the cab driver. B: $30,000 from the truck driver, and then the truck driver will be entitled to collect $10,000 from the cab driver. C: $30,000 from the truck driver, and then the truck driver will be entitled to collect nothing from the cab driver. D: nothing from the truck driver, because his percentage of fault is not greater than that of the plaintiff

C: $30,000 from the truck driver, and then the truck driver will be entitled to collect nothing from the cab driver. ---- The facts tell you that joint and several liability has been abolished, which means that each defendant is liable only for his own share of the damages, not the entire award. In addition, contribution between tortfeasors is only available where one defendant has paid more than his determined share of the damages

A car owner washed her car while it was parked on a public street, in violation of a local ordinance that prohibits the washing of vehicles on public streets during specified hours. The ordinance was enacted only to expedite the flow of automobile traffic. Due to sudden and unexpected cold weather, the car owner's waste water formed a puddle that froze in a crosswalk. A pedestrian slipped on the frozen puddle and broke her leg. The pedestrian sued the car owner to recover for her injury. At trial, the only evidence the pedestrian offered as to negligence was the car owner's admission that she had violated the ordinance. At the conclusion of the evidence, both parties moved for a directed verdict. How should the trial judge proceed? A: Deny both motions and submit the case to the jury, because, on the facts, the jury may infer that the car owner was negligent. B: Deny both motions and submit the case to the jury, because the jury may consider the ordinance violation as evidence that the car owner was negligent. C: Grant the car owner's motion, because the pedestrian has failed to offer adequate evidence that the car owner was negligent. D: Grant the pedestrian's motion, because of the car owner's admitted ordinance violation.

C: Grant the car owner's motion, because the pedestrian has failed to offer adequate evidence that the car owner was negligent.

A company designed and built a processing plant for the manufacture of an explosive chemical. An engineer was retained by the company to design a filter system for the processing plant. She prepared an application for a permit to build the plant's filter system and submitted it to the state's Department of Environmental Protection (DEP). As required by DEP regulations, the engineer submitted a blueprint to the DEP with the application for permit. The blueprint showed the entire facility and was signed and sealed by her as a licensed professional engineer. After the project was completed, a portion of the processing plant exploded, injuring the plaintiff. During discovery in an action by the plaintiff against the engineer, it was established that the explosion was caused by a design defect that was unrelated to the filter system designed by the engineer. However, the defect was present in the blueprint signed by the engineer. In that action, will the plaintiff prevail? A: Yes, because the engineer signed, sealed, and submitted a blueprint that showed the design defect. B: Yes, because all of the plant's designers are jointly and severably liable for the defect. C: No, because the engineer owed no duty to the plaintiff to prevent the particular risk of harm. D: No, because the engineer was an independent contractor

C: No, because the engineer owed no duty to the plaintiff to prevent the particular risk of harm.

A trucking company employed nine salaried dispatchers to ensure that its truck fleet operated according to schedule. Two years ago, as a cost-saving measure, the company reduced the number of dispatchers to six, and each of the remaining dispatchers had to work substantially longer hours. One of the remaining dispatchers complained to his supervisor that the stress and fatigue associated with the new working conditions were too much for him to handle. The supervisor told the dispatcher that he should quit if he couldn't handle the increased hours. Over the next three months, the dispatcher continued to complain about the working conditions, to no effect. The dispatcher suffered severe emotional distress from the working conditions, but no physical injury. He eventually was hospitalized and had to miss several months of work as a result of the emotional distress. The dispatcher sued the trucking company for negligence. The company has moved for summary judgment, based on the undisputed facts set out above. Assume that there is no applicable workers' compensation statute. How should the court rule on the motion? A: Deny the motion, because the jury must determine the extent of the emotional distress suffered by the dispatcher. B: Deny the motion, because there is evidence from which a jury could reasonably conclude that the supervisor failed to act with ordinary care. C: Grant the motion, because the dispatcher suffered no physical injury. D: Grant the motion, because there is no evidence from which a jury could reasonably conclude that the supervisor acted carelessly with respect to the dispatcher's emotional well-being.

C: Grant the motion, because the dispatcher suffered no physical injury. ---- As a general rule, a claim for negligently caused emotional distress must be connected to a physical injury. Here, the facts stipulate that the dispatcher suffered severe emotional distress but no physical injury.

While driving his pickup truck with a friend riding in the open bed, the driver swerved, throwing his friend to the pavement. The friend sustained severe injuries. The friend had often ridden in the open bed of the truck, and on some of those occasions the driver had swerved to frighten his friend. The friend sued the driver to recover both compensatory damages for his injuries and punitive damages. Which cause of action would NOT permit the friend to recover punitive damages? A: Assault. B: Battery. C: Negligence. D: Recklessness

C: Negligence. ---- Punitive damages are generally not recoverable in negligence cases unless additional proof is offered that the defendant was wanton and willful, reckless, or malicious, which was not the case here.

A company operates a factory that requires the use of very high voltage electricity. A neighbor owns property adjacent to the factory where he has attempted to carry on a business that requires the use of sensitive electronic equipment. Occasionally, the effectiveness of the neighbor's electronic equipment is slightly impaired by electrical interference arising from the high voltage currents used in the company's factory. The neighbor has complained to the company several times, with no result. There is no way that the company, by taking reasonable precautions, can avoid the interference with the neighbor's operation that arises from the high voltage currents necessary to the company's operation. In the neighbor's action against the company to recover damages for the economic loss caused to him by the electrical interference, will the neighbor prevail? A: Yes, because the company's activity is abnormally dangerous. B: Yes, for loss suffered by the neighbor after the company was made aware of the harm its activity was causing to the neighbor. C: No, because the company did not cause a substantial and unreasonable interference with the neighbor's business. D: No, because the neighbor's harm was purely economic and did not arise from physical harm to his person or property.

C: No, because the company did not cause a substantial and unreasonable interference with the neighbor's business. ---- The scope of interference is personal discomfort to the occupants or tangible harm to property, resulting in a diminution of its market value. Nuisance does not require proof of negligence; it just requires actual damages. In this case, the harm to the business was only occasional slight interference.

A hiker, although acting with reasonable care, fell while attempting to climb a mountain and lay unconscious and critically injured on a ledge that was difficult to reach. The plaintiff, an experienced mountain climber, was himself seriously injured while trying to rescue the hiker. The plaintiff's rescue attempt failed, and the hiker died of his injuries before he could be reached. The plaintiff brought an action against the hiker's estate for compensation for his injuries. In this jurisdiction, the traditional common-law rules relating to contributory negligence and assumption of risk remain in effect. Will the plaintiff prevail in his action against the hiker's estate? A: Yes, because his rescue attempt was reasonable. B: Yes, because the law should not discourage attempts to assist persons in helpless peril. C: No, because the hiker's peril did not arise from his own failure to exercise reasonable care. D: No, because the plaintiff's rescue attempt failed and therefore did not benefit the hiker.

C: No, because the hiker's peril did not arise from his own failure to exercise reasonable care.

A driver was driving his car near a homeowner's house when the homeowner's child darted into the street in front of the driver's car. As the driver swerved and braked his car to avoid hitting the child, the car skidded up into the homeowner's driveway and stopped just short of the homeowner, who was standing in the driveway and had witnessed the entire incident. The homeowner suffered from serious emotional distress from witnessing the danger to his child and to himself. Neither the homeowner nor his property was physically harmed. If the homeowner asserts a claim for damages against the driver but is unable to establish that the driver was negligent, will the homeowner still be able to prevail? A: Yes, because the driver's entry onto the homeowner's land was unauthorized. B: Yes, because the homeowner suffered serious emotional distress by witnessing the danger to his child and to himself. C: No, because the homeowner failed to show that the driver was negligent. D: No, because the homeowner's child was not exercising reasonable care.

C: No, because the homeowner failed to show that the driver was negligent. --- A claim for damages based solely on emotional distress requires at least negligence by the defendant in order to prevail

A mother rushed her eight-year-old daughter to the emergency room at a local hospital after the child fell off her bicycle and hit her head on a sharp rock. The wound caused by the fall was extensive and bloody. The mother was permitted to remain in the treatment room, and held the child's hand while the emergency room physician cleaned and sutured the wound. During the procedure, the mother said that she was feeling faint and stood up to leave the room. While leaving the room, the mother fainted and, in falling, struck her head on a metal fixture that protruded from the emergency room wall. She sustained a serious injury as a consequence. If the mother sues the hospital to recover damages for her injury, will she prevail? A: Yes, because the mother was a public invitee of the hospital's. B: Yes, because the fixture was not an obvious, commonly used, and essential part of the hospital's equipment. C: No, because there is no evidence that the hospital's personnel failed to take reasonable steps to anticipate and prevent the mother's injury. D: No, because the hospital's personnel owed the mother no affirmative duty of care

C: No, because there is no evidence that the hospital's personnel failed to take reasonable steps to anticipate and prevent the mother's injury.

The plaintiff, a jockey, was seriously injured in a race when another jockey, the defendant, cut too sharply in front of her without adequate clearance. The two horses collided, causing the plaintiff to fall to the ground, sustaining injury. The State Racetrack Commission ruled that, by cutting in too sharply, the defendant committed a foul in violation of racetrack rules requiring adequate clearance for crossing lanes. The plaintiff has brought an action against the defendant for damages in which one count is based on battery. Will the plaintiff prevail on the battery claim? A: Yes, if the defendant was reckless in cutting across in front of the plaintiff's horse. B: Yes, because the State Racetrack Commission determined that the defendant committed a foul in violation of rules applicable to racing. C: No, unless the defendant intended to cause impermissible contact between the two horses or apprehension of such contact by the plaintiff. D: No, because the plaintiff assumed the risk of accidental injury inherent in riding as a jockey in a horse race

C: No, unless the defendant intended to cause impermissible contact between the two horses or apprehension of such contact by the plaintiff.

A bright 12-year-old child attended a day-care center after school. The center was located near a man-made duck pond on the property of a corporation. During the winter, the pond was used for ice-skating when conditions were suitable. At a time when the pond was obviously only partially frozen, the child sneaked away from the center's property and walked out onto the ice over the pond. The ice gave way, and the child fell into the cold water. He suffered shock and would have drowned had he not been rescued by a passerby. At the time of the incident, the pond was clearly marked with numerous signs that stated, "THIN ICE—KEEP OFF." When the child sneaked away from the day-care center, the center was staffed with a reasonable number of qualified employees, and the employees were exercising reasonable care to ensure that the children in their charge did not leave the premises. There had not been a previous instance of a child coming onto the corporation's property from the day-care center. The jurisdiction follows a rule of pure comparative negligence. In a suit brought on the child's behalf against the day-care center and based only on the facts above, who is likely to prevail? A: The child, because he left the center while he was under the center's care. B: The child, because the day-care center is located near a pond. C: The day-care center, because it was not negligent. D: The day-care center, because the child was a trespasser.

C: The day-care center, because it was not negligent. ---- the facts state that the pond in question was the property of the corporation. Thus, the attractive nuisance doctrine would not apply under these facts against the day-care center.

A recently installed elevator suddenly started free-falling down the elevator shaft while carrying passengers. Frightened, a passenger pried the inside doors open and impulsively stuck his arm through them to try to stop the fall. As a result, his arm was broken. The elevator eventually stopped without causing further injuries. In a negligence action brought by the injured passenger against the company that installed and maintained the elevator, the injured passenger has asked the trial judge to instruct the jury that it may find the company negligent on a theory of res ipsa loquitur. In response, the company has argued that the passenger's conduct caused his injuries. How should the judge rule? A: The judge should deny the passenger's request, because it is possible that the company was not negligent. B: The judge should deny the passenger's request, because the jury could find that the conduct of the passenger contributed to his injuries. C: The judge should grant the passenger's request but should also instruct the jurors to consider any carelessness of the passenger in awarding damages if they find the company liable. D: The judge should grant the passenger's request, because the passenger acted reasonably considering the stress of the situation.

C: The judge should grant the passenger's request but should also instruct the jurors to consider any carelessness of the passenger in awarding damages if they find the company liable.

A man and his friend, who were both adults, went to a party. The man and the friend had many drinks at the party and became legally intoxicated. They decided to play a game of chance called "Russian roulette" using a gun loaded with one bullet. As part of the game, the man pointed the gun at the friend and, on her command, pulled the trigger. The man shot the friend in the shoulder. The friend has brought a negligence action against the man. Traditional defenses based on plaintiff's conduct apply. What is likely to be the dispositive issue in this case? A: Whether the game constituted a joint venture. B: Whether the friend could validly consent to the game. C: Whether the friend was also negligent. D: Whether the man was legally intoxicated when he began playing the game

C: Whether the friend was also negligent. ---- Contributory negligence is an appropriate defense to a negligence action, and here both parties seem to have been acting unreasonably in exactly the same way.

A customer bought a can of corn at a grocery store. While eating the corn later that evening, the customer was injured by a small piece of glass in the corn. The customer sued the canning company that had processed and canned the corn. At trial, the customer presented evidence that neither the customer nor any third party had done anything after the can of corn was opened that would account for the presence of the glass. Without any other evidence, is the customer likely to prevail? A: No, because it is possible that someone tampered with the can before the customer bought it. B: No, because the customer has not shown any direct evidence that the canning company acted negligently. C: Yes, because a jury may reasonably infer that the canning company acted negligently. D: Yes, because the grocery store could not have discovered the piece of glass by reasonable inspection.

C: Yes, because a jury may reasonably infer that the canning company acted negligently.

A man's car sustained moderate damage in a collision with a car driven by a woman. The accident was caused solely by the woman's negligence. The man's car was still drivable after the accident. Examining the car the next morning, the man could see that a rear fender had to be replaced. He also noticed that gasoline had dripped onto the garage floor. The collision had caused a small leak in the gasoline tank. The man then took the car to a mechanic, who owns and operates a body shop, and arranged with the mechanic to repair the damage. During their discussion the man neglected to mention the gasoline leakage. Thereafter, while the mechanic was loosening some of the damaged material with a hammer, he caused a spark, igniting vapor and gasoline that had leaked from the fuel tank. The mechanic was severely burned. The mechanic has brought an action to recover damages against the man and woman. The jurisdiction has adopted a pure comparative negligence rule in place of the traditional common law rule of contributory negligence. The jury found that while a reasonable person in the man's position would have warned the mechanic about the gasoline leak, the man had no actual knowledge of the risk that the gasoline leak presented. In this action, will the mechanic obtain a judgment against the man? A: No, because it was the mechanic's job to inspect the vehicle and repair whatever needed repair. B: No, because the man was not aware of the risk that the gasoline leak presented. C: Yes, because a reasonable person in the man's position would have warned the mechanic about the gasoline leak. D: Yes, because the car was unreasonably dangerous when the man delivered it to the mechanic.

C: Yes, because a reasonable person in the man's position would have warned the mechanic about the gasoline leak.

A security guard, dressed in plain clothes, was working for a discount store when a customer got into a heated argument with a cashier over the store's refund policy. Without identifying himself as a security guard, the security guard suddenly grabbed the customer's arm. The customer attempted to push the security guard away, and the security guard knocked the customer to the floor, causing injuries. The customer sued the discount store for battery on a theory of vicarious liability for the injuries caused by the security guard. The store filed an answer to the customer's complaint, asserting the affirmative defense of contributory negligence. The customer has moved to strike the affirmative defense. Traditional rules of contributory negligence apply. Should the trial court grant the customer's motion? A: No, because contributory negligence is an affirmative defense to a cause of action based on vicarious liability. B: No, because the customer should have known that his argument with the cashier might provoke an action by a security guard. C: Yes, because contributory negligence is not a defense to battery. D: Yes, because the customer did not know that he was pushing away someone who was employed as a security guard.

C: Yes, because contributory negligence is not a defense to battery. ---- Contributory negligence is a defense to negligence. Here, the action is for battery, an intentional tort. Therefore, the defense will not be available.

A consumer became physically ill after drinking part of a bottle of soda that contained a large decomposed snail. The consumer sued the store from which she had bought the soda to recover damages for her injuries. The parties agreed that the snail had been put into the bottle during the bottling process, over which the store had no control. The parties also agreed that the snail would have been visible in the bottle before the consumer opened it. Will the consumer be likely to prevail in an action against the store? A: No, because the consumer could have seen the snail in the bottle before she drank out of it. B: No, because the store was not responsible for the bottling process. C: Yes, because the consumer was injured by a defective product sold to her by the store. D: Yes, because the store had exclusive control over the bottle before selling it to the consumer.

C: Yes, because the consumer was injured by a defective product sold to her by the store. --- The seller of a product with a manufacturing defect that makes the product dangerous to the health of a consumer is strictly liable for the injuries caused by the defect.

A manufacturer entered into a five-year contract with a distributor to supply the distributor with products to be distributed in State A. The manufacturer reserved the right to enter into additional such contracts with other State A distributors. The manufacturer knew that the distributor would, on the basis of the agreement, invest in an expansion of its distribution centers in State A in order to handle the distribution of the manufacturer's products to retail sellers in that state. At the time the manufacturer entered into the contract, it had already firmly decided to withdraw from the State A market in two years. Because the distributor had heard rumors of such a plan, its chief executive officer, before signing the contract, asked a representative of the manufacturer whether such a plan was in place. The representative denied the existence of any such plan. Two years into the contract, the manufacturer announced that it no longer intended to sell products in State A and canceled its contract with the distributor. The manufacturer did not thereafter supply products for distribution or sale in State A. Does the distributor have a viable fraud claim against the manufacturer? A: No, because the distributor's exclusive remedy is for breach of contract. B: No, because the manufacturer did not misstate any material facts when entering into the contract. C: Yes, because the manufacturer denied that it had a withdrawal plan when the distributor asked about it at the time the contract was formed. D: Yes, because the manufacturer reserved the right to do business with other State A distributors

C: Yes, because the manufacturer denied that it had a withdrawal plan when the distributor asked about it at the time the contract was formed.

A plaintiff's three-year-old daughter was killed in an automobile accident. At the plaintiff's direction, the child's body was taken to a mausoleum for interment. Normally, the mausoleum's vaults are permanently sealed with marble plates secured by "tamper-proof" screws. After the child's body was placed in a mausoleum, however, only a fiberglass panel secured by caulking compound covered her vault. About a month later, the child's body was discovered in a cemetery located near the mausoleum. It had apparently been left there by vandals who had taken it from the mausoleum. As a result of this experience, the plaintiff suffered great emotional distress. If the plaintiff sues the mausoleum for the damages arising from her emotional distress, will she prevail? A: No, because the plaintiff experienced no threat to her own safety. B: No, because the mausoleum's behavior was not extreme and outrageous. C: Yes, because the mausoleum failed to use reasonable care to safeguard the body. D: No, because the plaintiff suffered no physical harm as a consequence of her emotional distress

C: Yes, because the mausoleum failed to use reasonable care to safeguard the body. ---- The mausoleum breached its duty of care regarding the child's body and the majority of courts allow standalone emotional harm to be recoverable where there has been a mishandling of a dead body of a relative resulting in severe emotional distress.

A patient who had suffered a severe fracture of her leg was treated by an orthopedist, who set the patient's leg and put it in a cast. When the leg continued to bother the patient six months later, she consulted a second orthopedist in the same town. The second orthopedist surgically inserted a pin to facilitate healing. The patient brought a malpractice action against the first orthopedist, claiming that he should have surgically inserted a pin at the time of initial treatment. The only evidence that the patient offered in support of her malpractice claim was the testimony of the second orthopedist, as follows: In response to the question "Would you have inserted a pin initially?" the second orthopedist testified, "I personally would not have been satisfied that the leg would heal properly without a pin." At the close of the patient's evidence, the first orthopedist moved for judgment as a matter of law. Should the motion be granted? A: No, because the patient has introduced evidence that the first orthopedist failed to give the care that the second orthopedist would have provided. B: No, because the second orthopedist practices in the same town and field of specialty as the first orthopedist. C: Yes, because the patient has failed to introduce evidence that the first orthopedist's care fell below the professional standard of care. D: Yes, because the second orthopedist also treated the patient and is thus not sufficiently objective to offer expert testimony.

C: Yes, because the patient has failed to introduce evidence that the first orthopedist's care fell below the professional standard of care.

A traveler was a passenger on a commercial aircraft owned and operated by an airline. The aircraft crashed into a mountain, killing everyone on board. The flying weather was good. The traveler's legal representative brought a wrongful death action against the airline. At trial, the legal representative offered no expert or other testimony as to the cause of the crash. On the airline's motion to dismiss at the conclusion of the legal representative's case, the court should A: grant the motion, because the legal representative has offered no evidence as to the cause of the crash. B: grant the motion, because the legal representative has failed to offer evidence negating the possibility that the crash may have been caused by mechanical failure that the airline could not have prevented. C: deny the motion, because the jury may infer that the aircraft crashed due to the airline's negligence. D: deny the motion, because in the circumstances common carriers are strictly liable.

C: deny the motion, because the jury may infer that the aircraft crashed due to the airline's negligence. ---- The evidence is sufficient to establish a res ipsa loquitur showing, which allows the trier of fact to infer the airline's negligence

While approaching an intersection with the red light against him, a motorist suffered a heart attack that rendered him unconscious. The motorist's car struck a child, who was crossing the street with the green light in her favor. Under the state motor vehicle code, it is an offense to drive through a red traffic light. The child sued the motorist to recover for her injuries. At trial it was stipulated that: (1) immediately prior to suffering the heart attack, the motorist had been driving within the speed limit, had seen the red light, and had begun to slow his car; (2) the motorist had no history of heart disease and no warning of this attack; (3) while the motorist was unconscious, his car ran the red light. On cross motions for directed verdicts on the issue of liability at the conclusion of the proofs, the court should A: grant the child's motion, because the motorist ran a red light in violation of the motor vehicle code. B: grant the child's motion, because, in the circumstances, reasonable persons would infer that the motorist was negligent. C: grant the motorist's motion, because he had no history of heart disease or warning of the heart attack. D: deny both motions and submit the case to the jury, to determine whether, in the circumstances, the motorist's conduct was that of a reasonably prudent person.

C: grant the motorist's motion, because he had no history of heart disease or warning of the heart attack.

A plaintiff and a man were passengers sitting in adjoining seats on a flight on an airline. There were many empty seats on the aircraft. During the flight, a flight attendant served the man nine drinks. As the man became more and more obviously intoxicated and attempted to engage the plaintiff in a conversation, the plaintiff chose to ignore the man. This angered the man, who suddenly struck the plaintiff in the face, giving her a black eye. If the plaintiff asserts a claim for damages against the airline based on battery, she will A: prevail, because she suffered an intentionally inflicted harmful or offensive contact. B: prevail, because the flight attendant acted recklessly in continuing to serve liquor to the man. C: not prevail, because the man was not acting as an agent or employee of the airline. D: not prevail, because she cannot establish some permanent injury from the contact

C: not prevail, because the man was not acting as an agent or employee of the airline. ---- For the airline to be held responsible for the man's commission of battery against the plaintiff, vicarious liabillity must apply. However, there is no evidence of a special relationship between the man and the airline, such as the man being an agent or employee. As such, vicarious liability will not attach

A patient had been under the care of a cardiologist for three years prior to submitting to an elective operation that was performed by a surgeon. Two days thereafter, the patient suffered a stroke, resulting in a coma, caused by a blood clot which formed after the operation. When it appeared that she had entered a permanent vegetative state, with no hope of recovery, the artificial life-support system that had been provided was withdrawn, and she died a few hours later. The withdrawal of artificial life support had been requested by her family, and duly approved by a court. The surgeon was not involved in that decision, or in its execution. The administrator of the patient's estate thereafter filed a wrongful death action against the surgeon, claiming that the surgeon was negligent in having failed to consult a cardiologist prior to the operation. At the trial the plaintiff offered evidence that accepted medical practice would require examination of the patient by a cardiologist prior to the type of operation that the surgeon performed. In this action, the plaintiff should A: prevail, because the surgeon was negligent in failing to have the patient examined by a cardiologist prior to the operation. B: prevail, because the blood clot that caused the patient's death was caused by the operation which the surgeon performed. C: not prevail, because there is no evidence that a cardiologist would have provided advice that would have changed the outcome if one had examined the patient before the operation. D: not prevail, because the surgeon had nothing to do with the withdrawal of artificial life support, which was the cause of the patient's death.

C: not prevail, because there is no evidence that a cardiologist would have provided advice that would have changed the outcome if one had examined the patient before the operation.

A patron ate a spicy dinner at a restaurant on Sunday night. He enjoyed the food and noticed nothing unusual about the dinner. Later that evening, the patron had an upset stomach. He slept well through the night, went to work the next day, and ate three meals. His stomach discomfort persisted, and by Tuesday morning he was too ill to go to work. Eventually, the patron consulted his doctor, who found that the patron was infected with a bacterium that can be contracted from contaminated food. Food can be contaminated when those who prepare it do not adequately wash their hands. The patron sued the restaurant for damages. He introduced testimony from a health department official that various health code violations had been found at the restaurant both before and after the patron's dinner, but that none of the restaurant's employees had signs of bacterial infection when they were tested one month after the incident. The restaurant's best argument in response to the patron's suit would be that A: no one else who ate at the restaurant on Sunday complained about stomach discomfort. B: the restaurant instructs its employees to wash their hands carefully and is not responsible if any employee fails to follow these instructions. C: the patron has failed to establish that the restaurant's food caused his illness. D: the patron assumed the risk of an upset stomach by choosing to eat spicy food.

C: the patron has failed to establish that the restaurant's food caused his illness. --- Health code violations can only establish duty and breach; they do not establish the restaurant's causal control over the specific instrumentality that caused the actual food poisoning. In addition, res ipsa loquitur does not apply because the patron was unable to show that the restaurant had exclusive control over everything the patron ate within the period leading up to his illness.

While driving his car, the plaintiff sustained injuries in a three-vehicle collision. The plaintiff sued the drivers of the other two vehicles, a truck and a bus, and each defendant crossclaimed against the other for contribution. The jurisdiction has adopted a rule of pure comparative negligence and allows contribution based upon proportionate fault. The rule of joint and several liability has been retained. The jury has found that the plaintiff sustained damages in the amount of $100,000, and apportioned the causal negligence of the parties as follows: The plaintiff 40%, the truck driver 30%, and the bus driver 30%. How much, if anything, can the plaintiff collect from the truck driver, and how much, if anything, can the truck driver then collect from the bus driver in contribution? A: Nothing, and then the truck driver can collect nothing from the bus driver. B: $30,000, and then the truck driver can collect nothing from the bus driver. C: $40,000, and then the truck driver can collect $10,000 from the bus driver. D: $60,000, and then the truck driver can collect $30,000 from the bus driver.

D: $60,000, and then the truck driver can collect $30,000 from the bus driver. --- Pure comparative negligence allows the plaintiff to recover all damages not attributed to his own negligence. The plaintiff is therefore entitled to $60,000, which is the $100,000 in damages he suffered minus his 40% of the fault. Since this is a joint and several liability jurisdiction, the defendants are each liable for the entire award.

While visiting at his son's home, a grandfather tripped on a toy left on the floor by his four-year- old grandson. The grandfather fell and was severely injured. The grandfather regularly visited his son's home and was aware that the grandson routinely left toys scattered about the house. The son had never warned the grandfather to look out for toys. The grandfather brought an action against his son to recover for his injuries, and both the grandfather and the son have moved for directed verdicts as to liability. The jurisdiction has abolished intra-family immunity and applies the traditional rules of landowner liability. What action should the court take? A: Deny both motions and submit the case to the jury based on negligence. B: Deny both motions and submit the case to the jury based on strict liability. C: Grant the grandfather's motion, because the son is liable as a matter of law for failing to warn about the risk of toys being left on the floor. D: Grant the son's motion, because the son had no duty to warn that the grandson might leave toys on the floor

D: Grant the son's motion, because the son had no duty to warn that the grandson might leave toys on the floor. --- Here, the grandfather was invited to the son's home, making him a licensee. Typically social guests are considered licensees. An owner owes a duty to warn or make safe dangerous known conditions that create an unreasonable risk of harm to the licensee and that the licensee is unlikely to discover. In this case, the grandfather regularly visits the son's home and was aware that toys were typically left around the house. The son had no duty to warn.

A surgeon performed a sterilization operation on a patient. After the surgery, the surgeon performed a test that showed that the patient's fallopian tubes were not severed, as was necessary for sterilization. The surgeon did not reveal the failure of the operation to the patient, who three years later became pregnant and delivered a baby afflicted with a severe birth defect that will require substantial medical care throughout its life. The birth defect resulted from a genetic defect unknown to, and undiscoverable by, the surgeon. The patient brought an action on her own behalf against the surgeon, seeking to recover the cost of her medical care for the delivery of the baby, and the baby's extraordinary future medical expenses for which the patient will be responsible. Which of the following questions is relevant to the lawsuit and currently most difficult to answer? A: Did the surgeon owe a duty of care to the baby with respect to medical services rendered to the patient three years before the baby was conceived? B: Can a person recover damages for a life burdened by a severe birth defect based on a physician's wrongful failure to prevent that person's birth from occurring? C: Did the surgeon owe a duty to the patient to inform her that the sterilization operation had failed? D: Is the patient entitled to recover damages for the baby's extraordinary future medical expenses?

D: Is the patient entitled to recover damages for the baby's extraordinary future medical expenses?

An intoxicated man who was standing on a fifth-floor apartment balcony threatened to jump off the building. A bystander pulled the man back into the building, pushed him into a bedroom, and locked the bedroom door from the outside. When the man became sober, the bystander released him from the bedroom. Does the man have a claim against the bystander? A: Yes, for battery, because the bystander pushed the man into the bedroom. B: Yes, for false imprisonment, because the bystander locked the man in the bedroom. C: Yes, for both battery and false imprisonment. D: No, because the bystander was privileged to act as he did

D: No, because the bystander was privileged to act as he did

The governor of a state signed a death warrant for a convicted murderer. Two protesters are active opponents of the death penalty. At a demonstration protesting the execution of the murderer, the protesters carried large signs that stated, "The governor - Murderer." A television station broadcasted news coverage of the demonstration, including pictures of the signs carried by the protesters. If the governor asserts against the television station a claim of damages for intentional infliction of emotional distress, will the governor prevail? A: Yes, because the broadcast showing the signs caused the governor to suffer severe emotional distress. B: Yes, because the assertion on the signs was extreme and outrageous. C: No, because the governor did not suffer physical harm as a consequence of the emotional distress caused by the signs. D: No, because the television station did not publish a false statement of fact with "actual malice."

D: No, because the television station did not publish a false statement of fact with "actual malice."

A landlord owns and operates a 12-story apartment building containing 72 apartments, 70 of which are rented. A pedestrian has brought an action against the landlord alleging that while he was walking along a public sidewalk adjacent to the landlord's apartment building a flower pot fell from above and struck him on the shoulder, causing extensive injuries. The action was to recover damages for those injuries. If the pedestrian proves the foregoing facts and offers no other evidence explaining the accident, will his claim survive a motion for directed verdict offered by the defense? A: Yes, because the pedestrian was injured by an artificial condition of the premises while using an adjacent public way. B: Yes, because such an accident does not ordinarily happen in the absence of negligence. C: No, because the landlord is in no better position than the pedestrian to explain the accident. D: No, because there is no basis for a reasonable inference that the landlord was negligent.

D: No, because there is no basis for a reasonable inference that the landlord was negligent. --- Any time negligence must be inferred, there is a res ipsa loquitur issue. For a claim based on res ipsa loquitur to prevail, the pedestrian must show that the landlord had exclusive control of the flowerpot before it fell.

A sporting goods shop was burglarized by an escaped inmate from a nearby prison. The inmate stole a rifle and bullets from a locked cabinet. The burglar alarm at the shop did not go off because the shop's owner had negligently forgotten to activate the alarm's motion detector. Shortly thereafter, the inmate used the rifle ammunition stolen from the shop in a shooting spree that caused injury to several people, including the plaintiff. If the plaintiff sues the shop's owner for the injury she suffered, will the plaintiff prevail? A: Yes, because the plaintiff's injury could have been prevented had the motion detector been activated. B: Yes, because the shop's owner was negligent in failing to activate the motion detector. C: No, because the storage and sale of firearms and ammunition is not an abnormally dangerous activity. D: No, because there is no evidence of circumstances suggesting a high risk of theft and criminal use of firearms stocked by the shop's owner.

D: No, because there is no evidence of circumstances suggesting a high risk of theft and criminal use of firearms stocked by the shop's owner. =--- The commission of a criminal act often supersedes the liability of the original negligent actor. An exception occurs if the negligent act creates a condition such that a criminal act is the foreseeable consequence of that action. Here, the shop owner didn't forget to lock his door; he just forgot to set the alarm on his gun shop

A blasting company was conducting blasting operations in connection with a highway-widening project. Prior to setting off a charge, the blasting company supervisor posted a large warning sign and stationed a flagman to stop automobiles along the highway. Although a motorist saw and understood both the sign and the flagman's instruction to stop, the motorist nonetheless continued past the flagman and the sign and was traveling along the highway at the moment of the blast. A flying rock from the blast hit and severely damaged the motorist's car. The jurisdiction follows the traditional common law rules governing contributory negligence and assumption of risk. If the motorist pursues a claim against the blasting company to recover for the damage to the car, and all the foregoing facts are established, how much should the motorist recover? A: The full value of the car. B: The full cost of restoring the car to its condition just before the accident. C: The full cost of restoring the car to its condition just before the accident, reduced by the percentage by which the motorist's conduct contributed to the accident. D: Nothing.

D: Nothing. ---- Under the traditional, common law rule of assumption of risk, a victim is barred from recovering damages if the injury results from the victim having knowingly and voluntarily chosen to encounter the risk of injury posed by the actor's conduct

A schizophrenic patient who was institutionalized in a psychiatric facility pushed a nurse down a stairwell at the facility. The nurse, a paid employee of the facility who was trained to care for schizophrenic patients, was injured. The patient is an indigent whose care is paid for by the government. The jurisdiction generally follows the rule that a person with a mental deficiency is held to the standard of a reasonable person. In a negligence action brought by the nurse against the patient, the patient's lawyer will argue that the patient should not be held responsible for the nurse's injury. Which of the following facts will be LEAST helpful to the patient's lawyer's argument? A: The nurse was a professional caregiver. B: The nurse was trained to care for patients with schizophrenia. C: At the time she pushed the nurse, the patient thought she was being attacked by an elephant. D: The patient is an indigent whose care is paid for by the government.

D: The patient is an indigent whose care is paid for by the government. ----

As a shopper was leaving a supermarket, an automatic door that should have opened outward opened inward, striking and breaking the shopper's nose. The owner of the building had installed the automatic door. The lease, pursuant to which the supermarket occupied the building, provided that the supermarket was responsible for all maintenance of the premises. The shopper sued the supermarket. At trial, neither the shopper nor the supermarket offered any testimony, expert or otherwise, as to why the door had opened inward. At the close of evidence, both the shopper and the supermarket moved for judgment as a matter of law. How should the trial judge rule? A: Grant judgment for the shopper, because it is undisputed that the door malfunctioned. B: Grant judgment for the supermarket, because the shopper failed to join the owner of the building as a defendant. C: Grant judgment for the supermarket, because the shopper failed to offer proof of the supermarket's negligence. D: Submit the case to the jury, because on these facts negligence may be inferred

D: Submit the case to the jury, because on these facts negligence may be inferred. ---- the jury could infer negligence because the accident occurred from a door in the exclusive control of the supermarket, and it is not the type of accident that would occur without negligence

A wealthy elderly woman was repeatedly harassed by a debt collector over a period of two months. The debt collector was trying to collect a large debt owed to his client by the woman's impoverished adult son. Although the debt collector knew that the woman was not legally responsible for the son's debt, he called the woman multiple times each day and threatened to destroy her credit. He also told her that he knew where she lived and that he was going to withdraw the money from her bank account. As a result, the woman suffered great mental anguish, was unable to sleep, and ultimately suffered serious health consequences. Which of the following conclusions would best support a claim by the woman against the debt collector for intentional infliction of emotional distress? A: The debt collector could reasonably have foreseen that the calls and threats might cause harm to the woman's health. B: The debt collector's conduct caused the woman to fear that he was someday going to physically attack her. C: The debt collector's conduct failed to comply with industry custom. D: The debt collector's conduct was extreme and outrageous

D: The debt collector's conduct was extreme and outrageous ---- the debt collector's conduct was extreme and outrageous, combined with the fact that the woman experienced severe emotional distress and that the debt collector was probably at least reckless with respect to the risk of causing her distress

A pedestrian was crossing a street in a crosswalk when a woman walking just ahead of him was hit by a truck. The pedestrian, who had jumped out of the way of the truck, administered CPR to the woman, who was a stranger. The woman bled profusely, and the pedestrian was covered in blood. The woman died in the ambulance on the way to the hospital. The pedestrian became very depressed immediately after the incident and developed physical symptoms as a result of his emotional distress. The pedestrian has brought an action against the driver of the truck for negligent infliction of emotional distress. In her defense, the driver asserts that she should not be held liable, because the pedestrian's emotional distress and resulting physical symptoms are not compensable. What is the strongest argument that the pedestrian can make in response to the driver's defense? A: The pedestrian saw the driver hit the woman. B: The pedestrian was acting as a Good Samaritan. C: The pedestrian was covered in the woman's blood and developed physical symptoms as a result of his emotional distress. D: The pedestrian was in the zone of danger.

D: The pedestrian was in the zone of danger. --- Because the pedestrian was in the path of the truck, he was under a direct physical threat from the driver's negligence. He could recover for the emotional distress that he suffered as a result of his fear for his own safety, and many courts would also allow him to recover for all other emotional distress that he suffered in connection with the event

A graduate student who was moving needed cardboard boxes, so she went to her usual grocery store to look for some. The store regularly gave repeat customers empty boxes to promote goodwill. Seeing no boxes outside, the student entered the store and asked a store employee for help. The employee pointed toward the rear of the store and said that all the empty boxes were in the storeroom. The student went into the storeroom through a door with a sign that said: "Keep Out. Employees Only." While in the storeroom, she tripped on a fold in a floor mat and fell into a stack of wooden crates. The topmost crate fell on the student, causing a head injury. The student has sued the store to recover for her injury. Which statement below is the most appropriate characterization of the student and her conduct under traditional common law rules? A: The student assumed the risk, because she knew that the storeroom was not normally accessible to the public. B: The student was a licensee in the store, because she had no intention of making a purchase at the store during the box-hunting visit. C: The student was a trespasser in the storeroom, because she ignored the sign on the door barring entrance to the storeroom. D: The student was an invitee in the storeroom, because she had permission to enter the storeroom consistent with the store's policy of making its empty boxes available to repeat customers.

D: The student was an invitee in the storeroom, because she had permission to enter the storeroom consistent with the store's policy of making its empty boxes available to repeat customers. ----- The store's policy, combined with the employee's express grant of permission, gave the student the status of invitee while she was in the storeroom looking for empty boxes.

The owner of a home in a rural area had for many years enjoyed unspoiled views of the surrounding countryside from her back deck. Several months ago, a neighboring farmer placed unsightly items, including an old, rusted tractor and some machine parts, entirely on his own property, but in a location visible from the homeowner's deck. The homeowner asked the farmer to move the items to a different area of the farm, out of the homeowner's line of sight. The farmer acknowledged that it was not common for farmers in the area to keep old equipment on their land in locations visible to neighbors, but nonetheless refused to move the items. Concerned that the farmer's placement of the items might adversely affect the resale value of the property, the homeowner paid for an appraisal of her own property. The appraisal determined that the market value of the property had not been diminished by the farmer's actions. If the homeowner were to sue the farmer for private nuisance, which of the following would be the farmer's best argument against liability? A: The unsightly items have not caused a decrease in the market value of the homeowner's property. B: The unsightly items do not physically encroach on the homeowner's property. C: It is not common for farmers in the area to keep old equipment on their land in places that are visible to neighbors. D: Unsightly conditions ordinarily do not of themselves amount to an unreasonable interference with the use and enjoyment of a neighboring property

D: Unsightly conditions ordinarily do not of themselves amount to an unreasonable interference with the use and enjoyment of a neighboring property --- Courts historically have held that causing a property owner to view unsightly things from their property does not by itself establish the sort of unreasonable interference with the use and enjoyment of property that is necessary for nuisance liability.

When a tire of a motorist's car suffered a blowout, the car rolled over and the motorist was badly injured. Vehicles made by the manufacturer of the motorist's car have been found to be negligently designed, making them dangerously prone to rolling over when they suffer blowouts. A truck driver who was driving behind the motorist when the accident occurred stopped to help. Rescue vehicles promptly arrived, and the truck driver walked along the side of the road to return to his truck. As he approached his truck, he was struck and injured by a speeding car. The truck driver has sued the manufacturer of the injured motorist's car. Is the truck driver likely to prevail in a suit against the car manufacturer? A: No, because the car manufacturer's negligence was not the proximate cause of the truck driver's injuries. B: No, because the truck driver assumed the risk of injury when he undertook to help the motorist. C: Yes, because it is foreseeable that injuries can result from rollovers. D: Yes, because the car manufacturer's negligence caused the dangerous situation that invited the rescue by the truck driver.

D: Yes, because the car manufacturer's negligence caused the dangerous situation that invited the rescue by the truck driver. --- By negligently designing the vehicles, the manufacturer became liable for foreseeable intervening causes of harm, including the invitation of rescue

An eight-year-old child went to the grocery store with her mother. The child pushed the grocery cart while her mother put items into it. The child's mother remained near the child at all times. Another customer in the store noticed the child pushing the cart in a manner that caused the customer no concern. A short time later, the cart the child was pushing struck the customer in the knee, inflicting serious injury. Assume that the child was negligent and the child's mother did not adequately supervise the child. If the customer brings an action, based on negligence, against the child's mother, will the customer prevail? A: Yes, because the child was negligent. B: Yes, because the child's mother is responsible for any harm caused by the child. C: Yes, because the child's mother assumed the risk of her child's actions. D: Yes, because the child's mother did not adequately supervise the child's actions.

D: Yes, because the child's mother did not adequately supervise the child's actions. --- Because the child's mother was not adequately supervising her daughter, and it was foreseeable that the child could potentially injure someone, the customer is likely to prevail.

A plaintiff entered a drug store to make some purchases. As he was searching the aisles for various items, he noticed a display card containing automatic pencils. The display card was on a high shelf behind a cashier's counter. The plaintiff saw a sign on the counter that read, "No Admittance, Employees Only." Seeing no clerks in the vicinity to help him, the plaintiff went behind the counter to get a pencil. A clerk then appeared behind the counter and asked whether she could help him. He said he just wanted a pencil and that he could reach the display card himself. The clerk said nothing further. While reaching for the display card, the plaintiff stepped sideways into an open shaft and fell to the basement, 10 feet below. The clerk knew of the presence of the open shaft, and had reason to believe that the plaintiff had not noticed it when stepping behind the counter. The plaintiff sued the drug store to recover damages for the injuries he sustained in the fall. The jurisdiction has adopted a rule of pure comparative negligence, and it follows traditional common-law rules governing the duties of a land possessor. Will the plaintiff recover a judgment against the drug store? A: No, because the plaintiff was a trespasser. B: No, because the plaintiff's injuries did not result from the defendant's willful or wanton misconduct. C: Yes, because the premises were defective with respect to a public invitee. D: Yes, because the clerk had reason to believe that the plaintiff was unaware of the open shaft.

D: Yes, because the clerk had reason to believe that the plaintiff was unaware of the open shaft. ---- As an invitee, and even as a trespasser once he went into the area of the store he was not permitted in, the plaintiff was owed a duty by the store, and by extension the clerk, to be warned of known, non-obvious, artificial dangers and for the premise to be made reasonably safe/

A 13-year-old girl was operating a high-speed motorboat. The boat was towing a 9-year-old boy in an inner tube tied to the rear of the motorboat by a rope. The rope became tangled around the boy's foot, causing him to suffer severe injuries. In a suit brought on the boy's behalf against the girl, the boy has introduced uncontroverted evidence that the girl drove carelessly in such a way as to entangle the boy in the rope. Is the boy likely to prevail? A: No, because the boy assumed the risk. B: No, because the girl was too young to be expected to appreciate and avoid the risk she exposed the boy to. C: Yes, because children of the girl's age should have the capacity to operate motorboats. D: Yes, because the girl will be held to an adult standard of care.

D: Yes, because the girl will be held to an adult standard of care.

A hot-air balloon touring company operated near a golf course. The company's property was separated from the golf course by a fence on which the company had posted signs warning people not to enter the property because of the dangers of balloons landing. A golfer on the golf course hit an errant shot onto the company's property, ignored the warning signs, and jumped over the fence to retrieve her golf ball. At about the same time, one of the company's balloons experienced mechanical problems and had to make an emergency landing to avoid crashing. The balloon, which was out of control when it landed, struck the golfer and injured her. The jurisdiction has decided that hot-air ballooning is an abnormally dangerous activity. In an action by the golfer against the company, does the company have any affirmative defenses? A: No, because the balloon was out of control when it struck the golfer. B: No, because the company was engaged in an abnormally dangerous activity. C: Yes, because the balloon landed to avoid crashing. D: Yes, because the golfer assumed the risk by coming onto the company's property

D: Yes, because the golfer assumed the risk by coming onto the company's property --- Despite the fact that this case involves an abnormally dangerous activity that would trigger strict liability, when the golfer ignored the visible warning signs and jumped over the fence voluntarily, she assumed the risk of being injured by hot-air balloons while on the company's property

A trucker driving down an isolated country road late one night struck cattle that had escaped from a farmer's pen and wandered into the road. The trucker was unable to stop before hitting the cattle but was not driving carelessly. While he was not injured in the collision, the trucker sustained damage to his truck and lost income during the time it took to repair the truck. The trucker sued the farmer for his damages and invoked the doctrine of res ipsa loquitur. At trial, the farmer introduced evidence that his cattle pen was of a sufficient height to prevent cattle from stepping over it and was constructed of thick steel pipe sitting in concrete with a substantial top rail. A sturdy pen such as this one would be more difficult for cattle to break through than one constructed of barbed wire or electric wire. Should the trial court allow the case to go to the jury with a res ipsa loquitur instruction? A: No, because it is possible that a third party wrongfully let the cattle out of the pen. B: No, because the trucker must submit direct evidence of negligence in order to invoke the res ipsa loquitur doctrine. C: Yes, because the farmer is strictly liable for harm caused by his escaping cattle. D: Yes, because the jury could conclude that cattle would not ordinarily escape a strong, secure cattle pen in the absence of negligence.

D: Yes, because the jury could conclude that cattle would not ordinarily escape a strong, secure cattle pen in the absence of negligence.

A man's car sustained moderate damage in a collision with a car driven by a woman. The accident was caused solely by the woman's negligence. The man's car was still drivable after the accident. Examining the car the next morning, the man could see that a rear fender had to be replaced. He also noticed that gasoline had dripped onto the garage floor. The collision had caused a small leak in the gasoline tank. The man then took the car to a mechanic, who owns and operates a body shop, and arranged with the mechanic to repair the damage. During their discussion the man neglected to mention the gasoline leakage. Thereafter, while the mechanic was loosening some of the damaged material with a hammer, he caused a spark, igniting vapor and gasoline that had leaked from the fuel tank. The mechanic was severely burned. The mechanic has brought an action to recover damages against the man and woman. The jurisdiction has adopted a pure comparative negligence rule in place of the traditional common law rule of contributory negligence. In this action, will the mechanic obtain a judgment against the woman? A: No, because there is no evidence that the woman was aware of the gasoline leak. B: No, because the mechanic would not have been harmed had the man warned him about the gasoline tank. C: Yes, because the mechanic was not negligent in failing to discover the gasoline leak himself. D: Yes, because the mechanic's injury was a proximate consequence of the woman's negligent driving.

D: Yes, because the mechanic's injury was a proximate consequence of the woman's negligent driving. --- It is foreseeable that a car accident could rupture a gas tank leading the gasoline to ignite and causing severe burn damage to anyone in or near the car, and the woman's negligence was the proximate cause of the accident and all foreseeable injuries

A fire that started in the defendant's warehouse spread to the plaintiff's adjacent warehouse. The defendant did not intentionally start the fire, and the plaintiff can produce no evidence as to how the fire started. However, the defendant had failed to install a sprinkler system, which was required by a criminal statute. The plaintiff can produce evidence that had the sprinkler system been installed, it could have extinguished the fire before it spread. In an action by the plaintiff against the defendant to recover for the fire damage, is it possible for the plaintiff to prevail? A: No, because the statute provides only for criminal penalties. B: No, because there is no evidence that the defendant negligently caused the fire to start. C: Yes, because a landowner is strictly liable for harm to others caused by the spread of fire from his premises under the doctrine of Rylands v. Fletcher. D: Yes, because the plaintiff was harmed as a result of the defendant's violation of a statute that was meant to protect against this type of occurrence.

D: Yes, because the plaintiff was harmed as a result of the defendant's violation of a statute that was meant to protect against this type of occurrence. ---- A criminal statute can be used to set the standard of care in a negligence action if it was intended to protect against the type of harm that occurred by specifying preventive steps that should be taken. In that case, violation of the statute is negligence per se.

While a woman was in her kitchen, she heard the screech of automobile tires. She ran to the window and saw a tricycle flying through the air. The tricycle had been hit by a car driven by a young man, who had been speeding. She also saw a child's body in the grass adjacent to the street. As a result of her shock from this experience, the woman suffered a heart attack. In a claim by the woman against the young man, the woman's right to recovery will depend on whether A: a person can recover damages based on the defendant's breach of a duty owed to another. B: it is foreseeable that a person may suffer physical harm caused solely by an injury inflicted on another. C: a person can recover damages caused by shock unaccompanied by bodily impact. D: a person can recover damages for harm resulting from shock caused solely by another's peril or injury.

D: a person can recover damages for harm resulting from shock caused solely by another's peril or injury. ---- an issue of negligent infliction of emotional distress. That claim is based on either being in the zone of danger created by the negligent defendant or witnessing a third party, the victim, be harmed. The woman was not in the zone of danger because she was inside.

An eight-year-old child went to the grocery store with her mother. The child pushed the grocery cart while her mother put items into it. The child's mother remained near the child at all times. Another customer in the store noticed the child pushing the cart in a manner that caused the customer no concern. A short time later, the cart the child was pushing struck the customer in the knee, inflicting serious injury. If the customer brings an action, based on negligence, against the grocery store, the store's best defense will be that A: a store owes no duty to its customers to control the use of its shopping carts. B: a store owes no duty to its customers to control the conduct of other customers. C: any negligence of the store was not the proximate cause of the customer's injury. D: a supervised child pushing a cart does not pose an unreasonable risk to other customers.

D: a supervised child pushing a cart does not pose an unreasonable risk to other customers. --- The facts in this question present no basis for holding the grocery store liable. This answer choice states that fact in the language of negligence law: the store simply was not acting negligently on the facts of this question because the child presented no unreasonable risk of harm to customers.

A plaintiff suffered a severe loss when his manufacturing plant, located in a shallow ravine, was flooded during a sustained rainfall. The flooding occurred because the city had failed to maintain its storm drain, which was located on city land above the plaintiff's premises, and because a railroad had failed to maintain its storm drain, which was located on railroad land below the plaintiff's premises. The flooding would not have occurred if either one of the two storm drains had been maintained properly. The plaintiff sued the railroad to recover compensation for his loss. The evidence in the case established that the failures of the two drains were caused by the respective negligence of the city and the railroad. There is no special rule insulating the city from liability. In his action against the railroad, the plaintiff should recover A: nothing, because he should have joined the city, without whose negligence he would have suffered no loss. B: nothing, because he did not introduce evidence that enables the court reasonably to apportion responsibility between the city and the railroad. C: one-half his loss, in the absence of evidence that enables the court to allocate responsibility fairly between the city and the railroad. D: all of his loss, because but for the railroad's negligence none of the flooding would have occurred.

D: all of his loss, because but for the railroad's negligence none of the flooding would have occurred. --- The "but for" test is satisfied where several acts combine to cause the injury, but none of the acts standing alone would have been sufficient. although the plaintiff's damages resulted from the negligence of two independent entities, neither of which alone would have been sufficient to flood the plaintiff's manufacturing plant, the combined negligence satisfies the but for test.

While a plaintiff was leaving an elevator, it suddenly dropped several inches, causing her to fall. An investigation of the accident revealed that the elevator dropped because it had been negligently maintained by an elevator company. The elevator company had a contract with the owner of the building to inspect and maintain the elevator. The plaintiff's fall severely aggravated a preexisting physical disability. If the plaintiff sues the elevator company for damages for her injuries, she should recover A: nothing, because the elevator company could not reasonably have been expected to foresee the extent of harm that the plaintiff suffered as a result of the accident. B: nothing, because the accident would not have caused significant harm to an ordinarily prudent elevator passenger. C: damages for the full amount of her disability, because a tortfeasor must take its victim as it finds her. D: damages for the injury caused by the falling elevator, including the aggravation of her preexisting disability.

D: damages for the injury caused by the falling elevator, including the aggravation of her preexisting disability.

A plaintiff and a man were passengers sitting in adjoining seats on a flight on an airline. There were many empty seats on the aircraft. During the flight, a flight attendant served the man nine drinks. As the man became more and more obviously intoxicated and attempted to engage the plaintiff in a conversation, the plaintiff chose to ignore the man instead of changing seats. This angered the man, who suddenly struck the plaintiff in the face, giving him a black eye. The flight attendant had witnessed the man becoming violent but chose to not get involved. If the plaintiff asserts a claim for damages against the airline based on negligence, the plaintiff will A: not recover, because a person is not required by law to come to the assistance of another who is imperiled by a third party. B: not recover, because the plaintiff could easily have moved to another seat. C: recover, because a common carrier is strictly liable for injuries suffered by a passenger while aboard the carrier. D: recover, because the flight attendants should have perceived the man's condition and acted to protect the plaintiff before the blow was struck.

D: recover, because the flight attendants should have perceived the man's condition and acted to protect the plaintiff before the blow was struck.


Conjuntos de estudio relacionados

Chapter 11 book notes (11.4, 11.5, 11.6)

View Set

Apex anesthesia: Autonomic Nervous System

View Set

Chapter 2 Test 1 Study Guide - Sharon Largarde

View Set

MTLE Pedagogy-Instruction and Assessment

View Set

Nutrition (Skills Module 3.0) Pretest (correct answers)

View Set